LSAT and Law School Admissions Forum

Get expert LSAT preparation and law school admissions advice from PowerScore Test Preparation.

 Administrator
PowerScore Staff
  • PowerScore Staff
  • Posts: 8937
  • Joined: Feb 02, 2011
|
#90596
Complete Question Explanation



Assumption. The correct answer choice is (E).

This stimulus has a classic LSAT structure - the professor presents a hypothesis that other people ("many scientists") believe, then the professor presents the evidence those scientists have for their hypothesis ("in support"). However, the professor does not think the hypothesis is as well-founded as those scientists think. The professor's conclusion is in the last sentence - there is no reason to believe the hypothesis, so the professor thinks. The only reason the professor gives is that the low visibility evidence that the scientists adduced for their conclusion is completely explained by general relativity.

It's important to note that the only support mentioned for the light-absorbing medium hypothesis is how it explains the low visibility of other star systems. The professor thinks the hypothesis has no basis because general relativity alone is enough to explain that low visibility, so there is no need to think a light-absorbing medium exists. One huge issue with this argument is that we have no idea what general relativity is! Because we aren't told what the general theory of relativity is, we can note several weaknesses in this argument. For one, if the theory of general relativity completely explains the low visibility, but the theory is otherwise defective, the argument doesn't look very good - one bad theory doesn't look like a good substitute for another bad theory. Another problem is that, because we don't know what the theory of general relativity is, it might actually hypothesize that there is a light-absorbing medium in space! If so, then general relativity is not a substitute for the scientists' hypothesis, but another version of their hypothesis.

These are problems with the argument. Because this is an Assumption question, the professor is assuming those problems don't exist. So we can prephrase that general relativity is at least an acceptable theory on other details, or that general relativity does not itself involve positing the existence of a light-absorbing medium in space.

Answer choice (A): This answer choice is tempting, but incorrect. The professor's argument does not think that the light-absorbing medium hypothesis does not explain the low visibility of other star systems. Instead the author thinks that, regardless of whether it explains that, general relativity also explains it, so we don't need to bother with the light-absorbing medium hypothesis.

Answer choice (B): As with answer choice (A), the issue is not whether the light-absorbing medium hypothesis explains the low visibility of other star systems. It may or may not do that. The professor doesn't care if it does - the professor thinks there's no need to bother with the hypothesis, as general relativity already provides a complete explanation.

Answer choice (C): The principle this answer choice expresses is entirely beside the point. The professor isn't trying to show that the light-absorbing medium hypothesis is incorrect. The professor is trying to show that there is no reason to believe it. The professor never actually states the hypothesis is wrong, but because some other theory provides a complete explanation, the light-absorbing medium hypothesis has no evidence for it. When this answer choice discusses whether a hypothesis is likely to be correct, that's an issue with which the professor does not deal, and thus which the professor does not have to assume.

Answer choice (D): The Assumption Negation Technique is a handy way to prove this answer choice incorrect. Assume this answer is false, so that its negation is true: "half or fewer of the scientists who posit the invisible light-absorbing medium accept the general theory of relativity." That negation is showing that the overlap between those accepting the light-absorbing medium hypothesis and those accepting general relativity is seemingly small. So what? That actually seems consistent with the professor's argument! If general relativity completely explains what the light-absorbing medium hypothesis was supposed to, then it's not surprising that few people would believe in both. But the Assumption Negation Technique says that the negation of the correct answer should be bad for the argument, not consistent with it. So this answer fails.

Answer choice (E): This is the correct answer choice. This answer matches one of our prephrases. The Assumption Negation Technique again looks like the best way to confirm that this is correct. If general relativity does depend on the hypothesis that there is a light-absorbing medium in space, then the professor is saying something like "The light-absorbing medium hypothesis has no basis because general relativity, a theory that ALSO posits such a medium, does a good enough job explaining things." This makes no sense. General relativity, according to the negation of this answer choice, also posits the medium, so there's no relevant difference between those theories. Thus, the author must assume this answer is true, as its falsity is fatal to the argument.
User avatar
 German.Steel
  • Posts: 55
  • Joined: Jun 12, 2021
|
#90976
I was scratching my head a little on (A). (E) is clearly and unambiguously correct so I wasn't overly troubled by it, but (A) is tempting nonetheless. Negated, it seems to say "this invisible nonsense does in fact explain adequately the existence of whatever." That struck me as hurting the argument, but I suppose ultimately not, because the argument takes the (kind of odd) stance that "it's irrelevant whether or not the invisible nonsense could explain it, because relativity already explains it. So even if the invisible nonsense would explain it, that's no reason to believe it."

Maybe my problem is with the argument itself more than with (A)! Seems dumb! If the invisible nonsense would adequately explain it, shouldn't we give it some consideration? Anyway, I'm rambling, lol.
 alicechoi86@gmail.com
  • Posts: 7
  • Joined: Jul 19, 2021
|
#91056
I mistakenly chose (D) here, instead of the correct answer (E).

I think I see now why (E) is correct. If the general theory of relativity DID depend upon the existence the medium, then it CANNOT act as the complete explanation of the low visibility. The medium's existence could then act as the explanation for the low visibility, in which case the Conclusion (NO REASON to believe the hypothesis) is untrue, and the argument breaks down.

I'm seeing now that (D) was incorrect b/c "most" is a problem in this case, since whether a majority of scientists who posit the hypothesis accept the general theory of relativity or not does NOT do anything to change the fact that the general theory of relativity already explains the low visibility given as a reason to support the hypothesis.

On review, I eliminated (A) via Assumption Negation:
low visibility COULD be adequately explained by the existence of a medium in outer space
(vs. the original answer: it WOULD NOT be adequately explained by the medium).
Neither of these affects the argument, b/c we already know that the general theory of relativity completely explains the low visibility. So, whether the medium in outer space DOES or DOES NOT explain the low visibility, we already have another explanation, so it would have no bearing on the argument.
 Adam Tyson
PowerScore Staff
  • PowerScore Staff
  • Posts: 5271
  • Joined: Apr 14, 2011
|
#91239
Excellent analysis by the both of you! And good job using the powerful Assumption Negation technique to see why those wrong answers are wrong. If those answers turn out to be false, the argument still works, so they are not necessary for the argument.

And yes, Alice, it doesn't matter what most of those scientists believe, or even what all of them believe. All that matters is that the theory of relativity doesn't depend on that invisible medium existing!
 ealanclos
  • Posts: 6
  • Joined: Mar 04, 2022
|
#95215
Hello,

After looking at it, I see why E is correct, but I originally chose C. Could someone please explain why c is incorrect? My thinking was that the author seems to think that since theory of relativity already explains the low-visibility of stars, the hypothesis must be incorrect. The part of answer choice C that says "not adequately accounted for by an existing theory" seemed to encapsulate this train of thought.
 Adam Tyson
PowerScore Staff
  • PowerScore Staff
  • Posts: 5271
  • Joined: Apr 14, 2011
|
#95240
Two problems come up with that answer choice as I see it, ealanclos. The first is that the author is not arguing that the theory is incorrect, but only that there is no reason to believe it. They are saying that since there already exists a theory that sufficiently explains it, there's no additional reason to bring in this new idea.

The second problem is one of conditional reasoning, those tricky "if this, then that" type of arguments. The author seems to assume that IF there is a theory that explains something, THEN there is no reason to believe an alternate hypothesis. The existence of the adequate theory is sufficient to dismiss the hypothesis.

Answer C makes what we call a Mistaken Negation of that conditional relationship. It's saying IF there is NOT a theory that explains it, THEN an alternate hypothesis should probably be accepted. But that's like saying "If I get into Yale Law School, I must have a good LSAT score, so if I do not get into Yale I must not have a good LSAT score." That's not necessarily true, because there could be many other reasons for my not getting into Yale, including that I never applied there!

Get the most out of your LSAT Prep Plus subscription.

Analyze and track your performance with our Testing and Analytics Package.